16
$\begingroup$

Is there any information known for the Ordinary Generating Function for Mobius? $$ \sum_{n=1}^{\infty} {\mu(n)}x^n $$ I know that

  1. It has radius of convergence 1.

  2. Does not have limit as $x\rightarrow 1$.

My question is

  1. Does it have limit as $x\rightarrow \textrm{exp}(i\theta)\neq 1$?

  2. Is there a similar result for $\textrm{exp}(i\theta)\neq 1$ with $$ \sum_{n\leq x}\mu(n)=o(x)$$ i.e. is there a result of this type? $$ \sum_{n\leq x}\mu(n)\textrm{exp}(in\theta) = o(x)$$

EDIT1: Aside from my questions, there are some known results for the OGF for Mobius.

EDIT2: Found a reference for (III)

Reference: Jason P. Bell, Nils Bruin, Michael Coons "Transcendence of generating functions whose coefficients are multiplicative" available at http://www.ams.org/journals/tran/2012-364-02/S0002-9947-2011-05479-6/

P. Borwein, T. Erd´elyi, and F. Littman, Polynomials with coefficients from a finite set available at http://www.cecm.sfu.ca/personal/pborwein/PAPERS/P195.pdf

(I) This function has the unit circle as natural boundary.

(II) This function is a transcendental function.

(III) It is not bounded on any open sector($\{z:|z|<1, z=re^{i\theta}, \alpha<\theta<\beta\}$)of unit disc.

This result (III) together with Baire Category answers my question 1 with a dense set of $\theta$'s, but as Prof Tao mentioned, there can be a possibility that it is bounded for some values of $\theta$.

$\endgroup$
4
  • $\begingroup$ As a side-remark: a Dirichlet-like regularization has been discussed here mathoverflow.net/q/47469/7710 $\endgroup$ Jan 30, 2018 at 11:14
  • $\begingroup$ I am interested in the statement 2, i. e. that it does not have a limit for $x \to 1$. Is there an easy way to see it? Graph of the function seems to suggets it has a finite limit at 1 (something like - 1). $\endgroup$
    – F. Jatpil
    Oct 29, 2021 at 18:32
  • $\begingroup$ We have $\sum_{n=1}^{\infty} \mu(n)e^{-n/x} = \Omega_{\pm} (\sqrt x)$ with the oscillation is about (in Python mpmath) >abs(gamma(zetazero(1))/zeta(zetazero(1),derivative=1)) This is approximately 7.197580271525181924896287e-10, it will indeed be hard to detect numerically. $\endgroup$ Oct 30, 2021 at 18:27
  • $\begingroup$ "I know that...Does not have limit as x→1". So what is the behavior of the function? It oscillates in the left proximity of +1? $\endgroup$
    – F. Jatpil
    Jan 13, 2023 at 13:04

2 Answers 2

14
$\begingroup$

Mobius randomness heuristics suggest that $\sum_n \mu(n) (r e^{i\theta})^n$ does not converge to a limit as $r \to 1^-$ for any (or at least almost any) $\theta$. If it did converge for some $\theta$, then we would have

$$\sum_n \mu(n) e^{in\theta} \psi_k(n) \to 0$$ as $k \to \infty$, where

$$ \psi_k(n) := (1-2^{-k-1})^n - (1-2^{-k})^n.$$

The cutoff function $\psi_k$ is basically a smoothed out version of the indicator function $1_{[2^k,2^{k+1}]}$. The Mobius randomness heuristic (discussed for instance in this article of Sarnak) then suggests that the sum $\sum_n \mu(n) e^{in\theta} \psi_k(n)$ should have a typical size of $2^{k/2}$, and so should not decay to zero as $k \to \infty$.

Note from Plancherel's theorem that the $L^2_\theta$ mean of $\sum_n \mu(n) e^{in\theta} \psi_k(n)$ is indeed comparable to $2^{k/2}$. This does not directly preclude the (very unlikely) scenario that this exponential sum is very small for many $\theta$ and only large for a small portion of the $\theta$, but if one optimistically applies various versions of the Mobius randomness heuristic (with square root gains in exponential sums) to guess higher moments of $\sum_n \mu(n) e^{in\theta} \psi_k(n)$ in $\theta$, one is led to conjecture a central limit theorem type behaviour for the distribution of this quantity (as $\theta$ ranges uniformly from $0$ to $2\pi$), i.e. it should behave like a complex gaussian with mean zero and variance $\sim 2^k$, and in particular it should only be $O(1)$ about $O(2^{-k})$ of the time, and Borel-Cantelli then suggests divergence for almost every $\theta$ at least. Unfortunately this is all very heuristic, and it seems difficult with current technology to unconditionally rule out a strange conspiracy that makes $\sum_n \mu_n (re^{i\theta})^n$ bounded as $r \to 1$ for some specific value of $\theta$ (say $\theta = \sqrt{2} \pi$), though this looks incredibly unlikely to me. (One can use bilinear sums methods, e.g. Vaughan identity, to get some nontrivial pointwise upper bounds on these exponential sums when $\theta$ is highly irrational, but I see no way to get pointwise lower bounds, since $L$-function methods will not be available in this setting, and there may well be some occasional values of $\theta$ and $k$ for which these sums are actually small. But it is likely at least that the $\theta=0$ theory can be extended to rational values of $\theta$.)

$\endgroup$
0
10
$\begingroup$

For (2), the answer is yes - see for example this paper of Baker and Harman.

$\endgroup$
3
  • $\begingroup$ Thank you for this. Actually their paper is conditional. Indeed, unconditional result by H. Davenport is qjmath.oxfordjournals.org/content/os-8/1/313.full.pdf. Anyway, that answers 2. $\endgroup$ Apr 3, 2013 at 21:01
  • 1
    $\begingroup$ Fair enough, although on the first page it cites unconditional results as well. $\endgroup$ Apr 4, 2013 at 5:57
  • $\begingroup$ Yeah. They cited Davenport's result. I enjoyed reading it. $\endgroup$ Apr 4, 2013 at 18:27

Your Answer

By clicking “Post Your Answer”, you agree to our terms of service and acknowledge you have read our privacy policy.

Not the answer you're looking for? Browse other questions tagged or ask your own question.